But, as I said, you don't actually need the coordinates at all.

Click For Summary
To calculate |u+v+w| for vectors u, v, and w, where |u|=√2, |v|=√3, and u is perpendicular to v, it is established that w=u×v has a magnitude of |w|=√6. The calculation of |u+v+w| can be approached without assuming specific vector coordinates by utilizing the properties of orthogonality. By expanding the square of the vector sum, it is shown that all inner products are zero due to orthogonality, leading to the conclusion that |u+v+w|=√11. This method confirms that the result holds true regardless of the coordinate system used.
hnnhcmmngs
Messages
19
Reaction score
0

Homework Statement



Calculate |u+v+w|, knowing that u, v, and w are vectors in space such that
|u|=√2, |v|=√3, u is perpendicular to v, w=u×v.

Homework Equations



|w|=|u×v|=|u|*|v|*sinΘ

The Attempt at a Solution


[/B]
Θ=90°
|w|=(√2)*(√3)*sin(90°)=√(6)

Then I tried to use
u={√2,0,0}
v={0,√3,0}
w={0,0,√6}
and I got that |u+v+w|=√(2+3+6)=√11
but I'm trying to find a way to do this problem where I don't assume that those are the vectors.
 
Physics news on Phys.org
hnnhcmmngs said:

Homework Statement



Calculate |u+v+w|, knowing that u, v, and w are vectors in space such that
|u|=√2, |v|=√3, u is perpendicular to v, w=u×v.

Homework Equations



|w|=|u×v|=|u|*|v|*sinΘ

The Attempt at a Solution


[/B]
Θ=90°
|w|=(√2)*(√3)*sin(90°)=√(6)

Then I tried to use
u={√2,0,0}
v={0,√3,0}
w={0,0,√6}
and I got that |u+v+w|=√(2+3+6)=√11
but I'm trying to find a way to do this problem where I don't assume that those are the vectors.

There is no loss of generality in what you did. Assuming that ##u,v,w## are vectors in some Cartesian coordinate system ##(x,y,z)##, just change coordinates to a new system ##(x', y', z')## in which ##u## points along the ##x'##-axis, ##v## points along the ##y'##-axis and ##w## points along the ##z'##-axis. That takes you right back to your original calculations.

Note added in edit: that argument assumes that the cross-product ##u \times v## behaves like a vector under a rotation; that is, if ##{\cal R}## is a rotation, then ##{\cal R}(u \times v) = {\cal R} u \: \times \: {\cal R} v.## (Here I mean that ##{\cal R} u## is the vector ##u## re-expressed in a new coordinate system obtained by applying ##{\cal R}## to the original unit coordinate vectors ##e_x, e_y, e_z##; it does not mean that the "physical" vector ##u## is rotated.) That "rotation" result is true, but needs to be proved, a task I will leave to you.
 
Last edited:
  • Like
Likes FactChecker
However, it should be noted that you can do it without ever referring to a coordinate system at all, just square and expand the vector sum:
$$
(\vec u + \vec v + \vec w)^2 = \vec u^2 + \vec v^2 + \vec w^2 + 2 (\vec u \cdot \vec v + \vec u \cdot \vec w + \vec v \cdot \vec w).
$$
Now, all of the inner products in the parenthesis are zero because all of the vectors are orthogonal. Furthermore
$$
\vec w^2 = (\vec u \times \vec v)^2 = (\vec u \times \vec v)\cdot(\vec u \times \vec v)
= -\vec v \cdot [\vec u \times (\vec u \times \vec v )].
$$
Apply the BAC-CAB rule:
$$
\vec w^2 = -\vec v \cdot [\vec u (\vec u \cdot \vec v)- \vec v (\vec u^2)] = \vec v^2 \vec u^2 = 6.
$$
It follows that
$$
(\vec u + \vec v + \vec w)^2 = 2+3+6 = 11.
$$
 
Question: A clock's minute hand has length 4 and its hour hand has length 3. What is the distance between the tips at the moment when it is increasing most rapidly?(Putnam Exam Question) Answer: Making assumption that both the hands moves at constant angular velocities, the answer is ## \sqrt{7} .## But don't you think this assumption is somewhat doubtful and wrong?

Similar threads

Replies
5
Views
2K
Replies
1
Views
1K
  • · Replies 2 ·
Replies
2
Views
2K
  • · Replies 16 ·
Replies
16
Views
2K
  • · Replies 9 ·
Replies
9
Views
5K
  • · Replies 2 ·
Replies
2
Views
2K
  • · Replies 6 ·
Replies
6
Views
11K
  • · Replies 4 ·
Replies
4
Views
2K
  • · Replies 4 ·
Replies
4
Views
6K
  • · Replies 2 ·
Replies
2
Views
2K